matheraum.de
Raum für Mathematik
Offene Informations- und Nachhilfegemeinschaft

Für Schüler, Studenten, Lehrer, Mathematik-Interessierte.
Hallo Gast!einloggen | registrieren ]
Startseite · Forum · Wissen · Kurse · Mitglieder · Team · Impressum
Forenbaum
^ Forenbaum
Status Schulmathe
  Status Primarstufe
  Status Mathe Klassen 5-7
  Status Mathe Klassen 8-10
  Status Oberstufenmathe
    Status Schul-Analysis
    Status Lin. Algebra/Vektor
    Status Stochastik
    Status Abivorbereitung
  Status Mathe-Wettbewerbe
    Status Bundeswettb. Mathe
    Status Deutsche MO
    Status Internationale MO
    Status MO andere Länder
    Status Känguru
  Status Sonstiges

Gezeigt werden alle Foren bis zur Tiefe 2

Navigation
 Startseite...
 Neuerdings beta neu
 Forum...
 vorwissen...
 vorkurse...
 Werkzeuge...
 Nachhilfevermittlung beta...
 Online-Spiele beta
 Suchen
 Verein...
 Impressum
Das Projekt
Server und Internetanbindung werden durch Spenden finanziert.
Organisiert wird das Projekt von unserem Koordinatorenteam.
Hunderte Mitglieder helfen ehrenamtlich in unseren moderierten Foren.
Anbieter der Seite ist der gemeinnützige Verein "Vorhilfe.de e.V.".
Partnerseiten
Weitere Fächer:

Open Source FunktionenplotterFunkyPlot: Kostenloser und quelloffener Funktionenplotter für Linux und andere Betriebssysteme
StartseiteMatheForenFolgen und GrenzwerteFolge
Foren für weitere Schulfächer findest Du auf www.vorhilfe.de z.B. Philosophie • Religion • Kunst • Musik • Sport • Pädagogik
Forum "Folgen und Grenzwerte" - Folge
Folge < Folgen+Grenzwerte < Analysis < Oberstufe < Schule < Mathe < Vorhilfe
Ansicht: [ geschachtelt ] | ^ Forum "Folgen und Grenzwerte"  | ^^ Alle Foren  | ^ Forenbaum  | Materialien

Folge: Frage (beantwortet)
Status: (Frage) beantwortet Status 
Datum: 19:46 Mi 01.11.2006
Autor: Phoney

Aufgabe
Berechne [mm] $\summe_{i=1}^{n}=i^3$ [/mm]

Guten Abend.

So einfach kann eine Aufgabe lauten, und trotzdem beiße ich mir die Zähne daran aus.

Ich erkenne einfach kein 'logisches' vorgehen, um nun eine Formel (mein TR vereinfacht es zu [mm] \br{n^2(n + 1)^2}{4} [/mm]

Wie kommt man auf so etwas?
Ich meine, wenn ich einfach mal ausprobiere

[mm] $\summe_{i=1}^{1}=i^3=1^3$ [/mm]
[mm] $\summe_{i=1}^{2}=i^3=1^3+2^3=9$ [/mm]
[mm] $\summe_{i=1}^{3}=i^3=9+3^3=9+27=36$ [/mm]
[mm] $\summe_{i=1}^{4}=i^3=36+64=100$ [/mm]

Ich erkenne da gar keinen Zusammenhang...
Wie berechne ich nun also: [mm] $\summe_{i=1}^{n}=i^3$? [/mm]

Gruß
Johann

        
Bezug
Folge: Antwort
Status: (Antwort) fertig Status 
Datum: 19:56 Mi 01.11.2006
Autor: Bastiane

Hallo Phoney,

> Berechne [mm]\summe_{i=1}^{n}=i^3[/mm]
>  Guten Abend.
>  
> So einfach kann eine Aufgabe lauten, und trotzdem beiße ich
> mir die Zähne daran aus.
>  
> Ich erkenne einfach kein 'logisches' vorgehen, um nun eine
> Formel (mein TR vereinfacht es zu [mm]\br{n^2(n + 1)^2}{4}[/mm]
>  
> Wie kommt man auf so etwas?
>  Ich meine, wenn ich einfach mal ausprobiere
>  
> [mm]\summe_{i=1}^{1}=i^3=1^3[/mm]
>  [mm]\summe_{i=1}^{2}=i^3=1^3+2^3=9[/mm]
>  [mm]\summe_{i=1}^{3}=i^3=9+3^3=9+27=36[/mm]
>  [mm]\summe_{i=1}^{4}=i^3=36+64=100[/mm]
>  
> Ich erkenne da gar keinen Zusammenhang...
>  Wie berechne ich nun also: [mm]\summe_{i=1}^{n}=i^3[/mm]?

guck doch mal []hier. ;-) Hab' einfach in google "Summe von Kubikzahlen" eingegeben. Nun hast du immerhin schon eine Formel und musst sie nur noch beweisen. Das dürfte mit vollständiger Induktion gehen.

Viele Grüße
Bastiane
[cap]

P.S.: Die Aufgabe dürftest du notfalls auch schon hier im Forum irgendwo finden. Und das Gleichheitszeichen nach dem Summenzeichn ist zu viel. ;-)

Bezug
        
Bezug
Folge: Mitteilung
Status: (Mitteilung) Reaktion unnötig Status 
Datum: 20:23 Mi 01.11.2006
Autor: ullim

Hi Phoney,

Du hast das Bildungsgesetz ja schon fast hingeschreiben.

es gilt für

n=1 [mm] \summe_{i=1}^{1}i^3=1=1^2 [/mm]          

n=2 [mm] \summe_{i=1}^{2}i^3=9=3^2 [/mm]          3=1+2

n=3 [mm] \summe_{i=1}^{3}i^3=36=6^2 [/mm]        6=1+2+3

n=4 [mm] \summe_{i=1}^{4}i^3=100=10^2 [/mm]   10=1+2+3+4

also für

n=N [mm] \summe_{i=1}^{N}i^3=(1+2+3+4+ [/mm] ... + [mm] N)^2=(\br{N(N+1)}{2})^2=\br{N^2(N+1)^2}{4} [/mm]

mfg ullim





Bezug
        
Bezug
Folge: Antwort
Status: (Antwort) fertig Status 
Datum: 21:04 Mi 01.11.2006
Autor: galileo

Hallo Phoney

Es gibt eine allgemeine Methode, wie man solche Summen berechnet. Man muss den Sumanden in der Form bringen:

[mm] \summe_{i=f}^{n}a_{i}=\summe_{i=f}^{n}(b_{i+1}-b_{i}) =b_{f+1}-b_{f}+b_{f+2}-b_{f+1}+b_{f+3}-b_{f+2} +\cdots +b_{n-1}-b_{n-2}+b_{n}-b_{n-1}+b_{n+1}-b_{n} =b_{n+1}-b_{f} [/mm]

Also, man muss [mm]a_{i}[/mm] splitten in der Form [mm]a_{i}=b_{i+1}-b_{i}[/mm]

Also, wir suchen [mm]b_{i}[/mm] in der Form
[mm] b_{i}=ai^{4}+bi^{3}+ci^{2}+di+e [/mm]
Wir müssen [mm]a,b,c,d,e[/mm] bestimmen. Wir haben die Bedingung:

[mm] i^{3}=\left( a(i+1)^{4}+b(i+1)^{3}+c(i+1)^{2}+d(i+1)+e\right) - \left( ai^4+bi^3+ci^2+di+e\right) [/mm]

Nach Umformung und Gleichsetzung der Koeffizienten erhalten wir:

[mm]a=1/4,\quad b=-1/2,\quad c=1/4,\quad d=e=0[/mm]

Also:

[mm]\summe_{i=1}^{n}i^3=\summe_{i=1}^{n}\left[ \left( \bruch{1}{4}(i+1)^4-\bruch{1}{2}(i+1)^3+\bruch{1}{4}(i+1)^2\right) -\left( \bruch{1}{4}i^4-\bruch{1}{2}i^3+\bruch{1}{4}i^2\right) \right] [/mm]

Und jetzt, substituierst in den teil mit (i+1) i durch n, und in den teil mit i, i durch 1.

[mm]\summe_{i=1}^{n}i^3= \bruch{1}{4}(n+1)^4-\bruch{1}{2}(n+1)^3+\bruch{1}{4}(n+1)^2 -\left( \bruch{1}{4}-\bruch{1}{2}+\bruch{1}{4}\right) =\bruch{1}{4}(n+1)^2\left( (n+1)^2-2(n+1)+1\right) = \bruch{n^2(n+1)^2}{4} [/mm]

Versuche es nachzuvollziehen! Wenn du etwas nicht verstanden hast, frage weiter.

Gruss galileo

Bezug
                
Bezug
Folge: Frage (beantwortet)
Status: (Frage) beantwortet Status 
Datum: 22:27 Mi 01.11.2006
Autor: Phoney

Moin Moin und ein ganz herzlichen Hallo, galileo.

> Versuche es nachzuvollziehen! Wenn du etwas nicht
> verstanden hast, frage weiter.

Nur gut, dass du das schreibst. Ansonsten hätte ich mich wahrscheinlich gar nicht noch einmal getraut, noch einmal nachzufragen, nach einer so super tollen Antwort. Vielen vielen Dank dafür!!

> Es gibt eine allgemeine Methode, wie man solche Summen
> berechnet. Man muss den Sumanden in der Form bringen:

Sehr sehr sehr schön. Nachdem ich die Mitteilung von ullim gelesen habe, die übrigens auch sehr gut ist, habe ich mich gefragt, ob ich dafür einfach nur ein gutes Auge brauche oder mir die Erfahrung es bringt, zu sehen, wie man auf die Vereinfachung kommt.
Normalerweise frustet mich diese Summenberechnung immer. Aber die Präsentation dieser Formel bringt mir ein Gefühl voller Freude!

> [mm] \summe_{i=f}^{n}a_{i}=\summe_{i=f}^{n}(b_{i+1}-b_{i}) =b_{f+1}-b_{f}+b_{f+2}-b_{f+1}+b_{f+3}-b_{f+2} +\cdots +b_{n-1}-b_{n-2}+b_{n}-b_{n-1}+b_{n+1}-b_{n} =b_{n+1}-b_{f} [/mm]
>  
> Also, man muss [mm]a_{i}[/mm] splitten in der Form
> [mm]a_{i}=b_{i+1}-b_{i}[/mm]

Das ist definitionsgemäß?

> Also, wir suchen [mm]b_{i}[/mm] in der Form
> [mm] b_{i}=ai^{4}+bi^{3}+ci^{2}+di+e [/mm]

Wie komme ich denn jetzt darauf, dass wir die Unbekannten a,b,c,d,e haben bzw. dass der höchste Exponent die vier ist? (Das hängt ja zusammen)

>  Wir müssen [mm]a,b,c,d,e[/mm] bestimmen. Wir haben die Bedingung:
>  
> [mm] i^{3}=\left( a(i+1)^{4}+b(i+1)^{3}+c(i+1)^{2}+d(i+1)+e\right) - \left( ai^4+bi^3+ci^2+di+e\right) [/mm]
>  
> Nach Umformung und Gleichsetzung der Koeffizienten erhalten
> wir:
>  
> [mm]a=1/4,\quad b=-1/2,\quad c=1/4,\quad d=e=0[/mm]
>  
> Also:
>  
> [mm]\summe_{i=1}^{n}i^3=\summe_{i=1}^{n}\left[ \left( \bruch{1}{4}(i+1)^4-\bruch{1}{2}(i+1)^3+\bruch{1}{4}(i+1)^2\right) -\left( \bruch{1}{4}i^4-\bruch{1}{2}i^3+\bruch{1}{4}i^2\right) \right] [/mm]

Bis hierhin wieder alles völlig logisch!

> Und jetzt, substituierst in den teil mit (i+1) i durch n,
> und in den teil mit i, i durch 1.

Das merke ich mir mal so.

> [mm]\summe_{i=1}^{n}i^3= \bruch{1}{4}(n+1)^4-\bruch{1}{2}(n+1)^3+\bruch{1}{4}(n+1)^2 -\left( \bruch{1}{4}-\bruch{1}{2}+\bruch{1}{4}\right) =\bruch{1}{4}(n+1)^2\left( (n+1)^2-2(n+1)+1\right) = \bruch{n^2(n+1)^2}{4} [/mm]

Ja, alles sonst logisch.

Hat die Methode eigentlich auch einen Namen?

Recht herzlichen Dank. Auch mit der vorherigen Erklärung kann ich schon viel anfangen!

Viele Grüße,
Johann

Bezug
                        
Bezug
Folge: Antwort
Status: (Antwort) fertig Status 
Datum: 22:56 Mi 01.11.2006
Autor: galileo

Danke, Phoney, für dein Feedback, habe mich sehr gefreut.

Bei einem Polinom n-ten Grades ist die Differenz vom Grad (n-1). Da wir [mm] i^3 [/mm] haben, ist n-1=3, also n=4.

Die Methode hat meines Wissens keinen Namen.

Lass uns den Spass an Mathe voll genießen!

Gruss galileo

Bezug
Ansicht: [ geschachtelt ] | ^ Forum "Folgen und Grenzwerte"  | ^^ Alle Foren  | ^ Forenbaum  | Materialien


^ Seitenanfang ^
www.schulmatheforum.de
[ Startseite | Forum | Wissen | Kurse | Mitglieder | Team | Impressum ]